in Calculus retagged by
901 views
3 votes
3 votes

The value of the infinite product

$$P=\frac{7}{9} \times \frac{26}{28} \times \frac{63}{65} \times \cdots \times \frac{n^3-1}{n^3+1} \times \cdots \text{ is }$$

  1. $1$
  2. $2/3$
  3. $7/3$
  4. none of the above
in Calculus retagged by
by
901 views

1 comment

$\frac{2}{3}$ ??
0
0

2 Answers

7 votes
7 votes
Best answer

Here, $\begin{align}P =\lim_{n\to \infty}\prod_{r=2}^{n} \frac{r^3-1}{r^3+1}&=\lim_{n\to \infty}\prod_{r=2}^{n} \left(\frac{r-1}{r+1}\right)\left(\frac{r^2+r+1}{r^2-r+1}\right)\\&=\lim_{n\to \infty}\left(\prod_{r=2}^{n} \frac{r-1}{r+1}\right) \left( \prod_{r=2}^{n} \frac{r^2+r+1}{r^2-r+1}\right)......\tag{i}\end{align}$

 

Now

$\begin{align} \prod_{r=2}^{n} \frac{r-1}{r+1}&=\frac{1}{3}\times\frac{2}{4}\times\frac{3}{5}\times\frac{4}{6}\times\frac{5}{7}\times\frac{6}{8}\times\cdots\times\frac{n-2}{n}\times\frac{n-1}{n+1}\\&=\frac{1\times2}{n(n+1)}; [\scriptsize \text{All numbers, except the first two numerators and the last two denominators,}\\&~~~~~~~~~~~~~~~~~~~~~~\scriptsize\text{ are crossed out by each other.}]\\&=\frac{2}{n(n+1)} \end{align}$

Then

$\begin{align} &~~~~\prod_{r=2}^{n} \frac{r^2+r+1}{r^2-r+1}\\&=\prod_{r=2}^{n} \frac{r(r+1)+1}{r(r-1)+1}\\&=\scriptsize\frac{(2\times3+1)}{(2\times1+1)}\times\frac{(3\times4+1)}{(3\times2+1)}\times\frac{(4\times5+1)}{(4\times3+1)}\times\cdots\times\frac{(n-1)n+1}{(n-1)(n-2)+1}\times\frac{n(n+1)+1}{n(n-1)+1}\\&=\frac{n(n+1)+1}{2\times1+1}; [\scriptsize \text{All numbers, except the first denominator and the last numerator,}\\&~~~~~~~~~~~~~~~~~~~~~~~~~~~~\scriptsize\text{ are crossed out by each other.}]\\&=\frac{n(n+1)+1}{3}  \end{align}$

 

Now putting these values to no$\mathrm{(i)}$, we get

$$\begin{align}P &=\lim_{n\to \infty}\left(\frac{2}{n(n+1)}\right) \left( \frac{n(n+1)+1}{3}\right)\\&=\frac{2}{3}\lim_{n\to \infty} \left( 1+\frac{1}{n(n+1)} \right)\\&=\frac{2}{3}; ~~[\because n\to \infty \Rightarrow \frac{1}{n}\to 0 \Rightarrow \frac{1}{n(n+1)}\to 0] \end{align}$$

 

So the correct answer is B.

selected by

1 comment

Nice answer.
1
1
3 votes
3 votes

Answer: $\mathbf B$

Given:$$P = \frac{7}{9} \times \frac{26}{28} \times \frac{63}{65}\times\dots\times\frac{n^3-1}{n^3+1}\times\dots$$

This can be written as,

$$\implies P_n \require {cancel} \require {circle}= \Big (\frac{1}{\color {red} 3} \times \frac{\cancel 7}{\cancel3}\Big)\times \Big(\frac{\color {red}2}{\cancel 4}\times\frac{\cancel{13}}{\cancel 7})\times\Big(\frac{\cancel 3}{\cancel 5}\times\frac{\cancel {21}}{\cancel {13}}\Big)\times\Big(\frac{\cancel4}{\cancel 7}\dots\times\Big(\frac{n-1}{n+1}\times \frac{n^2+n+1}{n^2-n+1}\Big)\;\;\;\;\;\;\;\;\;\;\;\;\;\;\scriptsize [\text{So, everything will cancel out except $2$ and $3$}]$$

$$\implies P_n =  \frac{2}{3} \times \frac{n^2+n+1}{n(n+1)} =\frac{2}{3}\times\Big(1 + \frac{1}{n(n+1)}\Big)$$

Now, as $$n \to \infty, \;P_n \to \frac{2}{3}$$

$\therefore$ the infinite product, $P$, converges to $\frac{2}{3}$; $\implies P_\infty = \frac{2}{3}$

$\therefore \; \mathbf B $ is the correct answer.

edited by
by
Quick search syntax
tags tag:apple
author user:martin
title title:apple
content content:apple
exclude -tag:apple
force match +apple
views views:100
score score:10
answers answers:2
is accepted isaccepted:true
is closed isclosed:true